Đến nội dung

Hình ảnh

CMR: $\frac{a+b+c}{3}\geq \sqrt[27]{\frac{a^3+b^3+c^3}{3}}$


  • Please log in to reply
Chủ đề này có 7 trả lời

#1
Baoriven

Baoriven

    Thượng úy

  • Điều hành viên OLYMPIC
  • 1425 Bài viết

Cho a,b,c thực dương thỏa mãn: $(a+b)(b+c)(c+a)=8$

Chứng minh rằng: $\frac{a+b+c}{3}\geq \sqrt[27]{\frac{a^3+b^3+c^3}{3}}$


$$\mathbf{\text{Every saint has a past, and every sinner has a future}}.$$


#2
doremon01

doremon01

    Trung sĩ

  • Thành viên
  • 153 Bài viết

BĐT cần CM tương đương với $(\frac{a+b+c}{3})^{27} \geq \frac{a^3+b^3+c^3}{3} $

Áp dụng bất đẳng thức Cauchy ta có:

$(\frac{a+b+c}{3})^{27}+8\geq 9\sqrt[9]{(\frac{a+b+c}{3})^{27}}=\frac{(a+b+c)^3}{3}\rightarrow (\frac{a+b+c}{3})^{27}\geq \frac{(a+b+c)^3-24}{3}=\frac{(a+b+c)^3-3(a+b)(b+c)(c+a)}{3}=\frac{a^3+b^3+c^3}{3}$ (đúng)



#3
Baoriven

Baoriven

    Thượng úy

  • Điều hành viên OLYMPIC
  • 1425 Bài viết

Cách Cauchy của bạn doremon01 là cách hay và đơn giản.

Một cách khác dùng đạo hàm.

Từ giả thiết ta được: $a+b+c\geq 3$

BĐT tương đương với: $(a+b+c)^27\geq 3^{26}[(a+b+c)^3-24]$

Đặt: $x=(a+b+c)^3\Rightarrow x\geq 27$ ta chứng minh $\frac{x^9}{x-24}\geq 3^{26}$

Xét hàm số $f(x)=\frac{x^9}{x-24},x\geq 27$ ta có:

$f'(x)=\frac{8x^8(x-27)}{(x-24)^2}\geq 0,\forall x\geq 27$

Do đó f(x) đồng biến trên $[27;+\infty )$

Vì vậy $f(x)\geq f(27)=3^{26}$

Đẳng thức xảy ra khi a=b=c=1


$$\mathbf{\text{Every saint has a past, and every sinner has a future}}.$$


#4
Nguyenngoctu

Nguyenngoctu

    Trung sĩ

  • Thành viên
  • 118 Bài viết

Nguyen Ngoc Tu inequality:

Cho a, b, c là các số thực không âm thỏa mãn $$ab + bc + ca = 3$$. Chứng minh rằng: $$\frac{{a + b + c}}{3} \ge \sqrt[n]{{\frac{{{a^2} + {b^2} + {c^2}}}{3}}},\,\,\forall n \ge 6,n \in N$$.


Bài viết đã được chỉnh sửa nội dung bởi Nguyenngoctu: 15-06-2016 - 20:40


#5
Nguyenngoctu

Nguyenngoctu

    Trung sĩ

  • Thành viên
  • 118 Bài viết

Taiwan, 2014:

Cho a, b, c là các số dương. Chứng minh rằng: $$3\left( {a + b + c} \right) \ge 8\sqrt[3]{{abc}} + \sqrt[3]{{\frac{{{a^3} + {b^3} + {c^3}}}{3}}}$$.


Bài viết đã được chỉnh sửa nội dung bởi Nguyenngoctu: 15-06-2016 - 20:40


#6
Nguyenngoctu

Nguyenngoctu

    Trung sĩ

  • Thành viên
  • 118 Bài viết

Cho a,b,c thực dương thỏa mãn: $(a+b)(b+c)(c+a)=8$

Chứng minh rằng: $\frac{a+b+c}{3}\geq \sqrt[27]{\frac{a^3+b^3+c^3}{3}}$

Lời giải gốc bài này có thể như sau:

Ta có:

$$\begin{array}{l} {\left( {a + b + c} \right)^3} = \sum {{a^3}} + 3\left( {a + b} \right)\left( {b + c} \right)\left( {c + a} \right) = \sum {{a^3}} + 24\\ = \sum {{a^3}} + \underbrace {3 + ... + 3}_{8\,\,la\`a n} \ge 9.\sqrt[9]{{\left( {{a^3} + {b^3} + {c^3}} \right){{.3}^8}}} = {3^3}.\sqrt[9]{{\frac{{{a^3} + {b^3} + {c^3}}}{3}}}\\ \Rightarrow {\left( {\frac{{a + b + c}}{3}} \right)^3} \ge \sqrt[9]{{\frac{{{a^3} + {b^3} + {c^3}}}{3}}} \Leftrightarrow \frac{{a + b + c}}{3} \ge \sqrt[{27}]{{\frac{{{a^3} + {b^3} + {c^3}}}{3}}}.\,\, \end{array}$$. Xong!



#7
Ankh

Ankh

    Hạ sĩ

  • Thành viên
  • 85 Bài viết

Nguyen Ngoc Tu inequality:

Cho a, b, c là các số thực không âm thỏa mãn $$ab + bc + ca = 3$$. Chứng minh rằng: $$\frac{{a + b + c}}{3} \ge \sqrt[n]{{\frac{{{a^2} + {b^2} + {c^2}}}{3}}},\,\,\forall n \ge 6,n \in N$$.

 Chú ý là với $n\geq 6$ thì $\left(\dfrac{a+b+c}{3}\right)^n\geq \dfrac{(a+b+c)^6}{729}=\dfrac{\left(\sum a^2+2\sum ab\right)^3}{729}\geq \dfrac{3\sum a^2.81}{729}$

 

 

Taiwan, 2014:

Cho a, b, c là các số dương. Chứng minh rằng: $$3\left( {a + b + c} \right) \ge 8\sqrt[3]{{abc}} + \sqrt[3]{{\frac{{{a^3} + {b^3} + {c^3}}}{3}}}$$.

 Sử dụng bất đẳng thức Holder thì $\text{VP}^3\leq 81\left(\dfrac{a^3+b^3+c^3}{3}+8abc\right)$

 Cho nên ta chỉ cần chứng minh $27(a+b+c)^3\geq 27(a^3+b^3+c^3)\geq 648abc\Leftrightarrow (a+b)(b+c)(c+a)\geq 8abc$

 Đúng theo bất dẳng thức AM-GM



#8
nguyenduy287

nguyenduy287

    Thượng sĩ

  • Thành viên
  • 256 Bài viết

Nguyen Ngoc Tu inequality:

Cho a, b, c là các số thực không âm thỏa mãn $$ab + bc + ca = 3$$. Chứng minh rằng: $$\frac{{a + b + c}}{3} \ge \sqrt[n]{{\frac{{{a^2} + {b^2} + {c^2}}}{3}}},\,\,\forall n \ge 6,n \in N$$.

 

Cho a,b,c thực dương thỏa mãn: $(a+b)(b+c)(c+a)=8$

Chứng minh rằng: $\frac{a+b+c}{3}\geq \sqrt[27]{\frac{a^3+b^3+c^3}{3}}$

ta sẽ chứng minh bài này bằng pp quy nạp:

Với n=6 bđt có dạng $\frac{\sum a}{3}\geq \sqrt[6]{\frac{\sum a^2}{3}}$

ta có $(a+b+c)^2=\sum a^2+6\geqslant 3\sqrt[3]{(\sum a^2)3^2}=9\sqrt[3]{(\sum a^2)\frac{1}{3}}$

suy ra $(\frac{\sum a}{3})^2\geq \sqrt[3]{\frac{\sum a^2}{3}}$

lấy căn ta có điều phải cm

với n lớn hơn 6 ,giả sử bđt đúng tới n=k tức là $(\frac{\sum a}{3})\geq \sqrt[k]{\frac{\sum a^2}{3}}$

ta sẽ chứng minh nó cũng đúng với n=k+1 

ta sẽ chứng minh bằng phản chứng như sau 

giả sử $\frac{\sum a}{3}< \sqrt[k+1]{\frac{\sum a^2}{3}}=(\frac{\sum a^2}{3})^{\frac{1}{k}-\frac{1}{k(K+1)}}\leq \frac{\sum a}{3}.(\frac{3}{\sum a^2})^{\frac{1}{k(K+1)}}$

$=>(\frac{\sum a^2}{3})^{\frac{1}{k(K+1)}}< 1=>\sum a^2< 3=ab+bc+ac$ ( vô lý ) 

nên ta có dpcm


Bài viết đã được chỉnh sửa nội dung bởi nguyenduy287: 15-06-2016 - 21:39

  "DÙ BẠN NGHĨ BẠN CÓ THỂ HAY BẠN KHÔNG THỂ, BẠN ĐỀU ĐÚNG "

                                                                                               -Henry Ford -

  

 

 

 

 





1 người đang xem chủ đề

0 thành viên, 1 khách, 0 thành viên ẩn danh